Difference between revisions of "2018 AMC 10A Problems/Problem 14"

(Solution 1)
Line 37: Line 37:
 
==Solution 3==
 
==Solution 3==
 
Let <math>x=\frac{3^{100}+2^{100}}{3^{96}+2^{96}}</math>. Multiply both sides by <math>(3^{96}+2^{96})</math>, and expand. Rearranging the terms, we get <math>3^{96}(3^4-x)+2^{96}(2^4-x)=0</math>. The left side is strictly decreasing, and it is negative when <math>x=81</math>. This means that the answer must be less than <math>81</math>; therefore the answer is <math>\boxed{(A)}</math>.
 
Let <math>x=\frac{3^{100}+2^{100}}{3^{96}+2^{96}}</math>. Multiply both sides by <math>(3^{96}+2^{96})</math>, and expand. Rearranging the terms, we get <math>3^{96}(3^4-x)+2^{96}(2^4-x)=0</math>. The left side is strictly decreasing, and it is negative when <math>x=81</math>. This means that the answer must be less than <math>81</math>; therefore the answer is <math>\boxed{(A)}</math>.
 +
 +
==Solution 4==
 +
A faster solution. Recognize that for exponents of this size <math>3^{n}</math> will be enormously greater than <math>2^{n}</math>, so the terms involving <math>2</math> will actually have very little effect on the quotient. Now we know the answer will be very close to <math>81</math>.
 +
 +
Notice that the terms being added on to the top and bottom are in the ratio <math>\frac{1}{16}</math> with each other, so they must pull the ratio down from 81 very slightly. (In the same way that a new test score lower than your current cumulative grade always must pull that grade downward.) Answer: <math>\boxed{(A)}</math>.
  
 
{{AMC10 box|year=2018|ab=A|num-b=13|num-a=15}}
 
{{AMC10 box|year=2018|ab=A|num-b=13|num-a=15}}

Revision as of 22:02, 8 February 2018

What is the greatest integer less than or equal to \[\frac{3^{100}+2^{100}}{3^{96}+2^{96}}?\]

$\textbf{(A) }80\qquad \textbf{(B) }81 \qquad \textbf{(C) }96 \qquad \textbf{(D) }97 \qquad \textbf{(E) }625\qquad$

Solution

Solution 1

Let's set this value equal to $x$. We can write \[\frac{3^{100}+2^{100}}{3^{96}+2^{96}}=x.\] Multiplying by $3^{96}+2^{96}$ on both sides, we get \[3^{100}+2^{100}=x(3^{96}+2^{96}).\] Now let's take a look at the answer choices. We notice that $81$, choice $B$, can be written as $3^4$. Plugging this into out equation above, we get \[3^{100}+2^{100} \stackrel{?}{=} 3^4(3^{96}+2^{96}) \Rightarrow 3^{100}+2^{100} \stackrel{?}{=} 3^{100}+3^4*2^{96}.\] The right side is larger than the left side because \[2^{100} \leq 2^{96}*3^4.\] This means that our original value, $x$, must be less than $81$. The only answer that is less than $81$ is $80$ so our answer is $\boxed{A}$.

~Nivek

Solution 2

Let $x=3^{96}$ and $y=2^{96}$. Then our fraction can be written as $\frac{81x+16y}{x+y}=\frac{16x+16y}{x+y}+\frac{65x}{x+y}=16+\frac{65x}{x+y}$. Notice that $\frac{65x}{x+y}<\frac{65x}{x}=65$. So , $16+\frac{65x}{x+y}<16+65=81$. And our only answer choice less than 81 is $\boxed{(A)}$

~RegularHexagon

Solution 3

Let $x=\frac{3^{100}+2^{100}}{3^{96}+2^{96}}$. Multiply both sides by $(3^{96}+2^{96})$, and expand. Rearranging the terms, we get $3^{96}(3^4-x)+2^{96}(2^4-x)=0$. The left side is strictly decreasing, and it is negative when $x=81$. This means that the answer must be less than $81$; therefore the answer is $\boxed{(A)}$.

Solution 4

A faster solution. Recognize that for exponents of this size $3^{n}$ will be enormously greater than $2^{n}$, so the terms involving $2$ will actually have very little effect on the quotient. Now we know the answer will be very close to $81$.

Notice that the terms being added on to the top and bottom are in the ratio $\frac{1}{16}$ with each other, so they must pull the ratio down from 81 very slightly. (In the same way that a new test score lower than your current cumulative grade always must pull that grade downward.) Answer: $\boxed{(A)}$.

2018 AMC 10A (ProblemsAnswer KeyResources)
Preceded by
Problem 13
Followed by
Problem 15
1 2 3 4 5 6 7 8 9 10 11 12 13 14 15 16 17 18 19 20 21 22 23 24 25
All AMC 10 Problems and Solutions